Laura Damone
Thanks Received: 94
Atticus Finch
Atticus Finch
 
Posts: 468
Joined: February 17th, 2011
 
 
 

Q15 - One should only buy a frying pan

by Laura Damone Fri Nov 06, 2020 5:47 pm

Question Type:
Strengthen

Stimulus Breakdown:
Conclusion: One should only buy a frying pan that has a warranty, even if it means paying more and even if you'd never bother to use the warranty if the product performed poorly or didn't last.

Premises: Manufacturers won't offer a warranty on a product if doing so means they'd have to reimburse a lot of folks because the product performed poorly or didn't last.

Answer Anticipation:
First, let's talk about that funky question stem. It's a strengthen question because you're trying to make the conclusion "strongly supported," but instead of saying "if which one of the following is true?" it says "if which one of the following is assumed?" Does that change how I'll prephrase or evaluate the answers? Not at all. I'm looking for the answer that will have the greatest positive impact on the argument. I'm not looking for an answer choice that describes an assumption the argument makes.

Ok, now that that's out of the way, let's prephrase! This seems like a pretty solid argument. The logic is essentially that a manufacturer won't offer a warranty on a product that won't perform well or won't last. If I don't see any major gaps in the argument, I think about defender strengtheners. Maybe my right answer will defend against some potential objection to buying only frying pans under warranty.

Correct answer:
A

Answer choice analysis:
(A) Hmm…At first, whether or not folks would actually seek reimbursement seemed irrelevant to me. It was only after eliminating all 5 answers that I came back for a closer look. When prephrasing, I understood the logic to be that a manufacturer won't offer a warranty on a product that won't perform well or won't last. But is that explicitly stated? Not quite. We know for sure that a manufacturer won't offer a warranty on a product if doing so means they'll have to reimburse a lot of people for poor performance or durability issues. But that's not the same as saying they won't offer a warranty on a product that will perform poorly or lack durability. We actually need to connect these two statements by asserting that a lot of folks would indeed seek reimbursement for these issues, and that's exactly what answer choice A does. Well played, LSAT...well played.

(B) Tempting! If we assume that all pans with warranties work at least as well as pans without warranties, that strengthens the argument that we should go with the warranty pans. It also defends against the potential objection that pans without warranties actually perform better. But the phrase "at the time of purchase" gave me pause. That's so narrow, and narrow isn't a feature you want in a Strengthen answer. I want my Strengthen answers to be widely applicable. So, even though I think this strengthens my argument a little, it doesn't quite strengthen it enough to make it a correct answer.

(C) This proportional relationship (more $ = more likely warranty) doesn’t help me choose the warranty pans over the no-warranty pans. Eliminate!

(D) The most expensive ones? That’s really narrow. Do we know whether the frying pans we're talking about are among the most expensive? Nope. Eliminate.

(E) Full customer satisfaction, eh? Sounds nice, so maybe it gives us one more little reason to go with warranty pans over no-warranty pans, but that's not enough to make it a correct answer in my book, in particular because it doesn't strengthen the connection between premise and conclusion.

Takeaway/Pattern:
When LSAT writers craft a correct answer that is particularly hard to ID as correct, that's hard enough. They can make the question even harder by making that correct answer answer choice A. I'm a huge advocate for working wrong to right, but sometimes that means we're so eager to eliminate that we get overzealous on the earliest answer choices. By the time we get to D and E with no contenders, we back off and start to look more closely. When this happens to you, resist the urge to give more lenience to D and E than you gave to A and B! If no answers look right, take a breath and dive back into A. Dig deep through the choices until you've found one you can really justify picking.

#officialexplanation
Laura Damone
LSAT Content & Curriculum Lead | Manhattan Prep